You are on page 1of 8

2014 Mock Paper (Compulsory Part) - Paper 2 (Full Solutions)

Paper 2
Full Solutions
6. B
Section A By the remainder theorem, we have
1. A f ( 1)  3
4n 1  4n 1  22( n 1)  22( n 1) (1) 2014  a ( 1)  b  3
2n 2 2n  2
2 2 1 a  b  3
2n 2 2n  2
2 ab  2
 24 n a  b  2
∴ 2a  2b  1
2. D  2(a  b)  1
1 y  2z  2(2)  1

x y  2z  3
y  2 z  x( y  2 z )
y  2 z  xy  2 xz 7. D
2 xz  2 z  xy  y x( x  k )  x  1
2 z ( x  1)  y ( x  1) x 2  kx  x  1
y ( x  1) x 2  (k  1) x  1  0
z
2( x  1) ∵ The equation has equal roots.
∴ 0
3. B ( k  1) 2  4(1)(1)  0
x2  x  y 2  y  ( x2  y 2 )  ( x  y) k 2  2k  1  4  0
 ( x  y )( x  y )  ( x  y ) k 2  2k  3  0
 ( x  y )( x  y  1) (k  3)(k  1)  0
k 3 0 or k  1  0
4. A k   3 or k 1
L.H.S.  A( x  3)( x  2)  Bx
 A( x 2  5 x  6)  Bx 8. C
 Ax  5 Ax  6 A  Bx
2 For I,
∵ The x-coordinate of P is smaller than 1.
 Ax 2  (5 A  B ) x  6 A
∴ a < 1, i.e. a > 1
By comparing the like terms, we have A  2 . ∴ I is true.
∴ 5(2)  B  6 For II,
10  B  6 ∵ The y-coordinate of the P is negative.
B4 ∴ b<0
Alternative solution: ∴ II is not true.
By substituting x = 2, we have For III,
A(2  3)(2  2)  B(2)  2(2)2  6(2)  12 y  ( x  a)2  b
 2B  8  x 2  2ax  a 2  b
∵ The y-intercept of the graph is smaller than 1.
B 4
∴ a2 + b < 1
∴ III is true.
5. D
∴ The answer is C.
3 x  5  8 and 9  x  3
3 x  3 and  x  6 9. B
x  1 and x 6 Let A, B and C be the heights of Albert, Betty and Cathy
∴ x6 respectively.
Then the heights of Albert and Betty is (1 + 20%)B and
(1  30%)C respectively.
i.e. A = 1.2B and B = 0.7C
A  1 .2 B
 1 . 2 ( 0 . 7 )C
 0.84C
 (1  16%)C
∴ Albert is shorter than Cathy by 16%.

1 © Pearson Education Asia Limited 2013


2014 Mock Paper (Compulsory Part) - Paper 2 (Full Solutions)
10. C 15. A
Jason still owes the bank just after his first instalment In △ ABC,
  5% 
12
 BC 2  AC 2  242  7 2
 $ 50 000  1    15 000
  12    576  49
 $37 558  625
Jason still owes the bank just after his second instalment  252
  5% 
12
  AB 2
 $37 558  1    15 000 ∴ △ ABC is a right-angled triangle, where
  12  
ACB = 90. (converse of Pyth. theorem)
 $24 480 (cor. to the nearest dollar) ∴ AB is a diameter of the circle.
(converse of  in semi-circle)
11. A Area of the shaded region
 area of semi-circle  area of △ ABC
2x  3y
1 2
 25  1 
x

3          24  7  cm 2
y 2  2  2  2 
∴ x:y=3:2  161 cm 2 (cor. to 3 sig. fig.)
x : y = 3 : 2

x : y = 32 : 22 16. B


Total surface area of the original cube  6a2 cm2
x : z = 23 : 13 Total surface area of 8 identical small cubes
∴ x : y : z = 6 : 4 : 3   a 2 
 8  6     cm 2
1 1
:
1 1 1 1
:  : :   2  

x y z 6 4 3
3a 2
 2:3: 4  8 cm 2
2
 12a 2 cm 2
12. A Percentage increase
The number of red cards in the box 12a 2  6a 2
3   100%
 40  6a 2
35  100%
 15
17. B
13. C In △ AEF and △ ADC,
∵ z varies directly as x and inversely as y . AFE = ACB (given)
kx EAF = DAC (common )
∴ z , where k is a non-zero constant. AEF = 180  AFE  EAF ( sum of △ )
y = 180   ACB   DAC
kx =  ADC
z
y ∴ △ AEF ~ △ ADC (AAA)
Let x cm2 be the area of △ AEF, then the area of △ ADC is
z y  kx (x + 21) cm2.
2
z y x  AF 
k , which is a constant.  
x x  21  AC 
1 x  4 
2
∴  must also be a constant.  
k z y
35
∴ The answer is C. 1

4
14. B 4 x  x  21
Lower limit of the actual length  (7  0.5) cm
x7
 6.5 cm ∴ The area of △ AEF is 7 cm2.
Lower limit of the actual width  (5  0.5) cm In △ ABF and △ AFE,
 4.5 cm
Area of △ ABF 8
The least possible perimeter of the rectangle 
 2  (6.5  4.5) cm Area of △ AFE 7
8
 22 cm Area of △ ABF   7 cm 2
7
 8 cm 2

2 © Pearson Education Asia Limited 2013


2014 Mock Paper (Compulsory Part) - Paper 2 (Full Solutions)
AF 4 2 21. C
 
AD 3  2 3 In △ BAP,
25 m

AF 2
 tan 38 
FD 1 AP
Area of △ ABF 2 25 m
 AP 
Area of △BDF 1 tan 38
 25 
1
Area of △BDF   8 cm 2 ∴ CP   40  m
2  tan 38 
 4 cm 2 In △ PCD,
∴ The area of △ ABC  (21  7  8  4) cm
2 CP
tan CDP 
CD
 40 cm 2
 25 
 40  m
 tan 38 
18. D CD 
∵ For 0  x  180 , 0  sin x  1 tan(90  76)
 32.1 m (cor. to the nearest 0.1 m)
The minimum value of 3  (2 sin x  1)
2

2
 1 
 3   2   1 22. D
 2  For I,
The sum of the interior angles of a regular hexagon
3
 (6  2)  180
12
∴ The required greatest value   720
3
∴ I must be true.
4
For II,
Each interior angle of a regular hexagon
19. A  720  6
∵ BC = BE = CD = DE
 120
∴ BCDE is a rhombus. Each exterior angle of a regular hexagon
∴ BCE = CED = 58 (property of rhombus)  360  6
ACB = 180  58  58 (adj. s on st. line)
 60
= 64
∴ Each interior angle is twice an exterior angle.
In △ ABC,
∵ AB = CB ∴ II must not be true.
For III,
∴ BAC  ACB (base s, isos. △ )
Regular hexagon has 6 axes of reflectional symmetry.
 64 ∴ III must be true.
ABC  180  64  64 ( sum of △ ) ∴ The answer is D.
 52
23. C
20. C Let AOB   .
Join OP. Consider the slope of L1, we have
Let POQ  x . tan  
a
ROQ  2POQ  2 x (arcs prop. to ∠s at centre) b
OQP  ROQ  2 x (alt. ∠s, PQ // OR) ∵ OA = AB
∴ ABO = AOB =  (base s, isos. △ )
In △ OQP,
∵ OQ = OP (radii) ∴ Slope of L2  tan(180   )
∴ OPQ = OQP = 2x (base s, isos. △ )   tan
∴ x  2 x  2 x  180 ( sum of △ )  a
   
x  36  b
1 a
RPQ   ROQ (∠at centre twice ∠ at ⊙ce) 
2 b
1
  2x
2 Alternative solution:
 36 Let (h, 0) be the coordinates of B.
∵ OA = AB
∴ The x-coordinate of A is the x-coordinate of the
mid-point of OB.

3 © Pearson Education Asia Limited 2013


2014 Mock Paper (Compulsory Part) - Paper 2 (Full Solutions)

h  For III,
Let  , k  be the coordinates of A. The equation of straight line passing through A and B:
2 
y  (  2 ) 2  1
h 
a   bk  0 x 1 1 2
2 y 2 3

ah
 bk  0 x 1 1
2 y  2  3x  3
ah  2bk 3x  y  5  0
2k a
  3 1
h b Substituting  ,   into 3x – y – 5 = 0,
2 2
k 0
The slope of L2  h 3  1
h L.H.S.  3       5
2 2  2

k 0
h
  R.H.S.
2 ∴ The centre of C lies on the straight line passing
2k
 through A and B.
h ∴ III is true.
a ∴ The answer is D.

b
26. C
24. A We can list all the possible outcomes in a table as follows:
Let (x, y) be the coordinates of P and F be a point on
Second card
x-axis such that PF is perpendicular to x-axis.
a b c d A B C D
PA  PF a a, b a, c a, d a, A a, B a, C a, D
( x  6) 2  [ y  (2)]2  y  0 b b, a b, c b, d b, A b, B b, C b, D
First card

c c, a c, b c, d c, A c, B c, C c, D
( x  6) 2  ( y  2) 2  y 2 d d, a d, b d, c d, A d, B d, C d, D
A A, a A, b A, c A, d A, B A, C A, D
x  12 x  36  y  4 y  4  y
2 2 2
B B, a B, b B, c B, d B, A B, C B, D
x 2  12 x  4 y  40  0 C C, a C, b C, c C, d C, A C, B C, D
D D, a D, b D, c D, d D, A D, B D, C
4 y   x  12 x  40
2

1 From the table, there are 56 possible outcomes altogether.


y   x 2  3x  10
4 There are 32 favourable outcomes.
32
P(only one cards marked a capital letter) 
25. D 56
For I, 4

2 x 2  2 y 2  6 x  2 y  13  0 7
13
x 2  y 2  3x  y  0
2 27. A
3 1 ∵ The median of the numbers is 6.
Centre   ,  
2 2 ∴ One of the unknowns is 6.
∴ I is not true. ∵ The mode of the numbers is 7.
For II, ∴ Another unknown is 7.
23367
2
 3   1   13 
2
The required mean   4.2
Radius            5
 2 2  2 
36
 28. A
4
For I,
3 Inter-quartile range of Class A = (60  30) marks
Distance between A and B = 30 marks
 (2  1) 2  [1  (2)]2 Inter-quartile range of Class B = (50  20) marks
= 30 marks
 1 9 ∴ I must be true.
 10 For II,
3 The lowest mark that students in Class B get in the test is
20 marks.
∴ II is true.
∴ II must be true.

4 © Pearson Education Asia Limited 2013


2014 Mock Paper (Compulsory Part) - Paper 2 (Full Solutions)
For III, For II,
∵ Box-and whisker diagram cannot show the number of Consider the slope of the graph, we have
students. 30
n
∴ III may not be true. 03
∴ The answer is A.  1
∴ II must not be true.
29. B For III,
Mean The relation between x and y is non-linear.
 12  18  21  21  21  23  24  25  ∴ III must not be true.
 
  27  29  32  32  36  38  40  ∴ The answer is A.
  cm  26.6 cm
15
  33. D
 
14F616  1  163  4  162  15  161  6  160
Standard deviation
 4096  1024  240  6
(12  26.6) 2  (18  26.6) 2  (21  26.6) 2 
 536610
(21  26.6) 2  (21  26.6) 2  (23  26.6) 2 
Dividing 5366 successively by 2, we have
(24  26.6) 2  (25  26.6) 2  (27  26.6) 2 
2 5366 remainder
(29  26.6) 2  (32  26.6) 2  (32  26.6) 2 
(36  26.6) 2  (38  26.6) 2  (40  26.6) 2 2 2683 …… 0
 cm
15 2 1341 …… 1
 7.60 cm (cor. to 2 d.p.)
2 670 …… 1
30. A 2 335 …… 0

2 167 …… 1
Section B
2 83 …… 1
31. D
2 41 …… 1
x x xy
 
x  y y  x x2  y2 2 20 …… 1
x x xy ……
   2 10 0
x  y  ( x  y) x 2  y 2
x( x  y ) x( x  y ) xy 2 5 …… 0
  
( x  y )( x  y ) ( x  y )( x  y ) x 2  y 2 2 2 …… 1
x  xy  x  xy  xy
2 2
 2 1 …… 0
x2  y2
 3 xy 0 …… 1
 2
x  y2 ∴ 536610  10100111101102
3 xy
 2
y  x2 34. B
∵ 2   2    3
32. A ∴  and  are the roots of the equation x2  x  3  0 .
y  mxn 1
∴     1
log 2 y  log 2 (mxn ) 1
3  3  3  
 log 2 m  log 2 x n
 n log 2 x  log 2 m  31
For I, 1

Consider the y-intercept of the graph, we have 3
log 2 m  3
m  23
8
∴ I must be true.

5 © Pearson Education Asia Limited 2013


2014 Mock Paper (Compulsory Part) - Paper 2 (Full Solutions)
35. A 37. C
1 1 Let S(n) = 2n2 – 5n.
 T(2014)
(1  i ) 2 1  2i  i 2
= S(2014) – S(2013)
1 = 2(2014)2 – 5(2014) – [2(2013)2 – 5(2013)]

1  2i  (1) = 8 102 322 – 8 094 273
1 = 8049
  I is true.
2i
T(1) = S(1) = 3
1 i
  For n  2,
2i i T(n) = S(n)  S(n  1)
i = (2n2  5n)  [2(n  1)2  5(n  1)]

2i 2 = (2n2  5n)  (2n2  4n + 2  5n + 5)
i = 4n  7

2 For n  1,
1 T(n + 1)  T(n) = [4(n + 1)  7]  (4n  7)
∴ The imaginary part is  . =4
2
 The sequence is an arithmetic sequence with common
difference 4.
36. D
 II is not true.
For III,
T ( n)  0
4n  7  0
7
n
4
∵ n is a positive integer.
∴ n=1
i.e. T(1) is the only negative term of the sequence.
∴ III is true.
∴ The answer is C.

From the graph, (1, 1), (4, 6) and (7, 0) lie in D. 38. B
∴ The answer is D. ∵ The vertical difference between the maximum point
and the minimum point of the graph is 4  0 = 4.
Alternative solution: ∴ The graph of y  a sin( x  30) has enlarged along the
For I, y-axis to 2 times that of y  sin( x  30) .
putting x = 1 and y = 1, ∵ The graph of y  a sin( x  30) is reflected along
x=10
the x-axis by comparing to that of y  sin( x  30) .
y=10
x  y = 1  1 = 0  2 ∴ a<0
3x + 2y = 3 + 2 = 5  24 i.e. a = 2
 (1, 1) lies in D. From the graph, when x = 300, y = .
For II, 4  2 sin(300  30)  b
putting x = 4 and y = 6, 4  2(1)  b
x=40 b2
y=60 ∴ The answer is B.
x  y = 4  6 = 2  2
3x + 2y = 12 + 12 = 24  24 39. C
 (4, 6) lies in D. With the notations in the figure, O is the projection of V
For III, on the plane ABCD.
putting x = 7 and y = 0, M is the mid-point of AB. E is a point on VB, such that
x=70 AE  VB and CE  VB.
y=00
x  y = 7  0 = 0  2
3x + 2y = 21 + 0 = 21  24
 (7, 0) lies in D.
 The answer is D.

6 © Pearson Education Asia Limited 2013


2014 Mock Paper (Compulsory Part) - Paper 2 (Full Solutions)
The required angle is∠AEC. ∴  of (*)  0
∵ ABCD is a square. [(6k  26)]2  4(10)(k 2  6k  15)  0
12
∴ OM  cm  6 cm 36k 2  312k  676  40k 2  240k  600  0
2
 4k 2  72k  76  0
VM  82  62 cm (Pyth.theorem)
k 2  18k  19  0
 10 cm
In △ VMB, (k  19)(k  1)  0
10  1  k  19
tan VBM  ∴ The answer is C.
6
VBM  59.0362435
In △ ABE, 42. D
For I,
AE
sin VBM  ∵ △ ABC is an isosceles triangle.
AB ∴ y-axis is the perpendicular bisector of AC.
AE  12 sin 59.0362435 cm ∴ The circumcentre of △ ABC lies on the y-axis.
 10.2899151 cm ∴ I must be true.
Similarly, For II,
CE  10.2899151 cm ∵ △ ABC is an isosceles triangle.
In △ ABC, ∴ y-axis is the median of △ ABC from B to AC.
AC  122  122 cm (Pyth.theorem) ∴ The centroid of △ ABC lies on the y-axis.
∴ II must be true.
 288 cm
For III,
In △ AEC, by the cosine formula, Let Y(0, y) be the circumcentre of △ ABC.
10.28991512  10.28991512  ( 288 ) 2 The circumcentre of △ ABC is the centre of the
cos AEC  circumcircle.
2(10.2899151)(10.2899151)
∵ BY = CY
AEC  111 (cor. to the nearest degree)
∴ The required angle is 111°. ∴ (9  y ) 2  (3  0) 2  (0  y ) 2
81  18 y  y 2  9  y 2
40. C
Join BC. 18 y  72
PBC  ADC (ext. , cyclic quad.) y4
 102 ∴ The radius of the circumcircle is 4.
In △PBC, ∴ III must be true.
PCB  180  102  26 (∠ sum of △) ∴ The answer is D.
 52
43. B
BAC  PCB (∠ in alt. segment)
The required number of ways  C14  2  2  P33  2
 52
 192
BAC  CAD  DAR  180 (adj. s on st. line)
52  2DAR  180
44. C
DAR  64 Let R, Y and B represent red ball, yellow ball and blue ball
In △ADR, respectively.
ARD  102  64 (ext. ∠ of △) P(exactly two draws)
 38  P(RR )  P(YY )  P(BB)
C24  C26  C25

41. C C215
 x 2  y 2  6 x  8 y  15  0 31
By solving  , we have 
x  3 y  k  0 105
P(three balls in different colours after three draws)
(3 y  k ) 2  y 2  6(3 y  k )  8 y  15  0
C 4  C15  C16
9 y 2  6ky  k 2  y 2  18 y  6k  8 y  15  0  1
C315
10 y 2  (6k  26) y  k 2  6k  15  0 ……(*) 24
∵ The circle and the line intersect at two distinct points. 
91

7 © Pearson Education Asia Limited 2013


2014 Mock Paper (Compulsory Part) - Paper 2 (Full Solutions)
The required probability
 1  P(exactly two draws) 
P(three balls in different colours after three draws)
31 24
1 
105 91
86

195

Alternative solution:
The required probability
 4 5 3 4 6 3
        2
 15 14 13 15 14 13 
 6 4 5 6 5 5
        2
 15 14 13 15 14 13 
 5 6 4 5 4 4
        2
 15 14 13 15 14 13 
44 18 40
  
455 91 273
86

195

45. D
∵ The extra datum in the group {m1, x + 1, x + 4, x + 7,
x +10} is the mean of the group {x + 1, x + 4, x + 7,
x +10}.
∴ m2 = m1
Consider the equation for finding the variance of the group
{m1, x + 1, x + 4, x + 7, x +10}.
(m2  m1 )2  ( x  1  m2 )2  ( x  4  m2 )2 
( x  7  m2 )2  ( x  10  m2 )2
v2 
5
∵ The numerator is the same as that of v1 while the
denominator is greater than that of v1.
∴ v1 > v2
∴ The answer is D.

Alternative solution:
By substituting x = 0, we have
1  4  7  10
m1   5.5
4
(1  5.5)2  (4  5.5)2  (7  5.5)2  (10  5.5)2
v1 
4
 11.25
1  4  5.5  7  10
m2   5.5
5
(1  5.5) 2  (4  5.5) 2  (5.5  5.5) 2 
(7  5.5) 2  (10  5.5) 2
v1 
5
9
∴ m1 = m2 and v1 > v2

8 © Pearson Education Asia Limited 2013

You might also like